Why is A correct? Why is D incorrect?
Thanks
Ryan-Mahabir on September 23, 2019
  • December 2016 LSAT
  • SEC4
  • Q6
1
Reply
Why would D be an unacceptable answer?
Assuming that the organ had been modified several times before it was destroyed during WWII, woul...
Brennen on September 19, 2019
  • December 2016 LSAT
  • SEC4
  • Q13
1
Reply
Help please
None of the answer choices appealed to me; can someone please explain? Thanks!
Minerva on August 26, 2019
  • December 2016 LSAT
  • SEC4
  • Q7
1
Reply
Why is D wrong?
Is D wrong because it doesn't say that Hai's offer was legitimate? And if it did say his offer wa...
ryan-fadden on May 30, 2019
  • December 2016 LSAT
  • SEC4
  • Q24
1
Reply
Can someone go through this please
Thank you
alymathieu on January 24, 2019
  • December 2016 LSAT
  • SEC4
  • Q20
1
Reply
Why is the answer B
and not D? D isn't the best answer but B didn't really seem to be the point - the flaw was that h...
GLEE on December 16, 2018
  • December 2016 LSAT
  • SEC4
  • Q26
1
Reply
Weaken
Hello, I picked the wrong answer. How do I go about seeing why the answer I chose is wrong an...
UTSmtihie on October 20, 2018
  • December 2016 LSAT
  • SEC4
  • Q25
4
Replies
Answer Choice C
Is C correct because it rules out the possibility that some other entity is funding the research?
RyanW on September 4, 2018
  • December 2016 LSAT
  • SEC4
  • Q10
1
Reply
(D) and (E)
Hello - having difficulties understanding the difference between (D) and (E). Seems like negating...
hannah93092 on May 26, 2018
  • December 2016 LSAT
  • SEC4
  • Q18
1
Reply